La ricerca ha trovato 61 risultati

da Leonida
16 set 2012, 18:10
Forum: Teoria dei Numeri
Argomento: Sulla reciprocità
Risposte: 8
Visite : 2092

Re: Sulla reciprocità

@Troleito: quello che dici mi sembra giusto. Questo problema sembra essere N5 del PreIMO 2011, solo che in quest'ultimo si chiedeva di dimostrare che $ \displaystyle p\mid 7^{\frac{p-1}{2}}+1 $ :D Forse Matty96 ha commesso un errore di battitura...
da Leonida
28 ago 2012, 20:28
Forum: Teoria dei Numeri
Argomento: [tex]f(n) \neq f(m)[/tex] se [tex]m-n \in \mathbb{P}[/tex]
Risposte: 2
Visite : 946

Re: [tex]f(n) \neq f(m)[/tex] se [tex]m-n \in \mathbb{P}[/te

Claim: $f(n) = \displaystyle \frac{n \cdot \varphi(n)}{2}$. Sia $S$ la sommatoria nel testo. Gli addendi che compaiono in $S$ sono proprio $\varphi(n)$. Inoltre per le proprietà del M.C.D, $(i,n) = (n-i, n)$; pertanto se $i$ compare nella sommatoria, allora compare anche $n-i$. Si può quindi vedere ...
da Leonida
04 ago 2012, 00:42
Forum: Teoria dei Numeri
Argomento: 121. C<f(n)<f(n+1)<...<f(n+99)<C^2
Risposte: 6
Visite : 1669

Re: 121. C<f(n)<f(n+1)<...<f(n+99)<C^2

Lemma : esistono almeno 100 numeri primi $p$ tali che $C < p < C^2$. Dim.: dimostro per induzione su $k$ che tra $C$ e $2^k C$ vi sono almeno $k$ numeri primi. Passo base : $k=1$. Segue dal il Teorema di Chebyshef (o Postulato di Bertrand), il quale afferma che esiste almeno un primo tra $C$ e $2C$...
da Leonida
03 ago 2012, 18:48
Forum: Teoria dei Numeri
Argomento: Disuguaglianza su $\varphi(\cdot)$
Risposte: 2
Visite : 889

Re: Disuguaglianza su $\varphi(\cdot)$

Nonostante il testo poco accattivante, ci provo :) Lemma degli Scout : se $a \mid b$ allora $\varphi(a) \mid \varphi(b)$. Dim.: deve valere $\displaystyle a= \displaystyle \prod_{i=1}^{k} q_i^{\alpha_{i}}$ e $\displaystyle b= \displaystyle \prod_{i=1}^{k} q_i^{\beta_{i}} \cdot B$, con $B \in \mathbb...
da Leonida
28 lug 2012, 20:18
Forum: Teoria dei Numeri
Argomento: 121. C<f(n)<f(n+1)<...<f(n+99)<C^2
Risposte: 6
Visite : 1669

Re: 121. C<lpf(n)<lpf(n+1)<lpf(n+2)<C^(3/2)

Ma almeno uno tra $ lpf(n) $, $ lpf(n+1) $ e $ lpf(n+2) $ è uguale a 2... Sbaglio?
da Leonida
22 lug 2012, 01:37
Forum: Teoria dei Numeri
Argomento: Dalle APMO 2003...
Risposte: 8
Visite : 2559

Re: Dalle APMO 2003...

Uhm, occhio ad alcuni passaggi: $\displaystyle s$ può anche essere 1! (se ho capito cosa è $\displaystyle s$)
da Leonida
21 lug 2012, 18:30
Forum: Teoria dei Numeri
Argomento: Dalle APMO 2003...
Risposte: 8
Visite : 2559

Re: Dalle APMO 2003...

Bene il punto (a) :)
Riguardo il tuo dubbio, attenzione: tu vuoi dimostrare che $n$ divide $(n -k)!$, non che $p$ divide $(n -k)!$. Nel tuo caso si ha $35 \mid 16!$
che è vero.
da Leonida
21 lug 2012, 17:00
Forum: Teoria dei Numeri
Argomento: Dalle APMO 2003...
Risposte: 8
Visite : 2559

Dalle APMO 2003...

Sia k \geq 14 un intero e sia \displaystyle p_k il più grande numero primo strettamente minore di k . Sia n un numero composto. Dimostrare che: (a) se \displaystyle n= 2p_k , allora n NON divide \displaystyle (n-k)! (b) se \displaystyle n> 2p_k , allora n divide \displaystyle (n-k)! (si può assumere...
da Leonida
21 lug 2012, 16:10
Forum: Matematica non elementare
Argomento: Un limite "diofantino"
Risposte: 14
Visite : 5366

Re: Un limite "diofantino"

@ant.py: occhio, gli $y$ adeguati possono essere \displaystyle \left\lfloor \frac{n}{ab}\right\rfloor oppure \displaystyle\left\lfloor \frac{n}{ab}\right\rfloor +1 . Ad esempio 2x +3y = 11 ha come soluzioni non negative (4,1) e (1,3) nonostante \left\lfloor \frac{11}{6}\right\rfloor = 1 . Comunque i...
da Leonida
17 lug 2012, 00:08
Forum: Teoria dei Numeri
Argomento: Dubbi binari
Risposte: 2
Visite : 961

Re: Dubbi binari

A me viene 2n, ma in realtà anche a te viene 2n! Guarda la seconda uguaglianza alla terzultima riga del tuo post: sottrai 1 da un numero di $ n+2 $ cifre e ottieni un numero di $ n+1 $ cifre??? C'è solo un caso in cui ciò può accadere :wink:
da Leonida
14 lug 2012, 11:37
Forum: Olimpiadi della matematica
Argomento: Stage Senior 2012
Risposte: 327
Visite : 95194

Re: Stage Senior 2012

Gli esercizi sono da spedire entro domenica. Vale come gli anni scorsi che è sufficente che loro se li trovino nella mail il lunedì mattina ? Bè, l'anno scorso la scadenza era il 15 Luglio ma molte persone (me compreso) avevano inviato gli esercizi la sera del 15... Analogamente se quest'anno la sc...
da Leonida
16 mag 2012, 19:09
Forum: Teoria dei Numeri
Argomento: Radici di tre..
Risposte: 2
Visite : 1262

Re: Radici di tre..

Riscrivo come $(n\sqrt{3}- \left\lfloor n\sqrt{3}\right\rfloor)\cdot n\sqrt{3} > 1 \longrightarrow 3n^{2} -n\sqrt{3}\left\lfloor n\sqrt{3}\right\rfloor -1 > 0$. Le radici dell'equazione di secondo grado associata sono $n_{1,2}= \displaystyle \frac{\sqrt{3}\left\lfloor n\sqrt{3}\right\rfloor \pm \sqr...
da Leonida
16 mag 2012, 18:58
Forum: Teoria dei Numeri
Argomento: Care vecchie diofantee
Risposte: 9
Visite : 3156

Re: Care vecchie diofantee

LeZ ha scritto: Mi sa che non è la strada più veloce anche perchè mi sto perdendo! Aiutini?
Concentrati su $2 \cdot 3^k = 5^y +1$! Hai trovato la soluzione con k=1, come puoi dimostrare che con k>1 non ci sono soluzioni?
da Leonida
26 apr 2012, 11:45
Forum: Olimpiadi della matematica
Argomento: BMO 2012
Risposte: 18
Visite : 7812

Re: BMO 2012

benzo494 ha scritto: devo dedurre che quest'anno i BMOisti saranno a Cesenatico? Niente più concomitanza?
Deduzione esatta, ed è un bene per alcune squadre :D
In bocca al lupo anche da parte mia!!
da Leonida
20 apr 2012, 19:22
Forum: Teoria dei Numeri
Argomento: Egmo
Risposte: 8
Visite : 2727

Re: Egmo

Uhm... Che mi dici di questa: $\frac {3}{4} + \frac {6}{5} = \frac {39}{20} = \frac {156}{80} = \frac {2 \cdot 78}{2 + 78}$?
Perchè il testo non dice che la frazione a destra deve essere ridotta ai minimi termini...